多元微积分作业(二)

本文最后更新于:2022年3月14日 晚上

微积分A2作业(二)

未央-能动12 罗诗棋 2021012525 王晓峰老师

1.3.5

(连续性与偏连续性) 设 \(f:[a, b] \times[c, d] \rightarrow \mathbb{R}\) 。 (a) 如果 \(f\) 是连续函数,那么是否成立:对任意 \(y \in[c, d], f(\cdot, y)\) : \([a, b] \rightarrow \mathbb{R}\) 连续;对任意 \(x \in[a, b], \quad f(x, \cdot):[c, d] \rightarrow \mathbb{R}\) 连续? (b) 如果对任意 \(y \in[c, d], \quad f(\cdot, y):[a, b] \rightarrow \mathbb{R}\) 连续;对任意 \(x \in[a, b]\), \(f(x, \cdot):[c, d] \rightarrow \mathbb{R}\) 连续,那么 \(f\) 是否为连续函数?考虑 \[ f(x, y)= \begin{cases}\dfrac{x y}{x^{2}+y^{2}}, & x y \neq 0 \\ 0, & x y=0 .\end{cases} \] (c) 设 \(x_{0} \in[a, b], y_{0} \in[c, d], f\) 满足: \(f\left(\cdot, y_{0}\right):[a, b] \rightarrow \mathbb{R}\) 连续;并且对任意 \(\varepsilon>0\), 存在 \(\delta(\varepsilon)>0\) 使得对任意 \(x \in[a, b]\) 以及任意 \(y \in[c, d]\) ,只要 \(\left|y-y_{0}\right|<\delta(\varepsilon)\), 就有 \(\left|f(x, y)-f(x, y_{0})\right|<\varepsilon\) 。证明 \(f\)\(\left(x_{0}, y_{0}\right)\) 连续。 (d) 设 \(f(x, y)\) 分别对每个变量 \(x\)\(y\) 是连续的,且对 \(y\) 是单调不减的,证明 \(f\) 关于 \((x, y)\) 是连续的。


解:

  1. \(f\) 是连续函数,对 \((x_0,y_0)\in[a, b] \times[c, d]\)\(\forall \varepsilon>0\)\(\exists \delta>0\) 使 \(\forall ||(x,y)-(x_0,y_0)||<\delta\) ,都有 \(|f(x,y)-f(x_0,y_0)|<\varepsilon\) 。令 \(|x-x_0|<\delta,y=y_0\) ,有 \(||(x,y)-(x_0,y_0)||\leq||(x,y)-(x_0,y_0)||_{\infty}<\delta\) ,因此 \(|f(x,y_0)-f(x_0,y_0)|<\varepsilon\)

故 $ f(, y):[a, b] $ 在 \([a,b]\) 是连续的。同理 \(f(x, \cdot):[c, d] \rightarrow \mathbb{R}\)\([c,d]\) 也是连续的。

  1. 对示例函数,若 \(x=0\)\(y=0\) ,函数值恒为 \(0\) ,连续;若 \(x\neq0\)\(y\neq0\) ,由于 \(\lim\limits_{x\to0,y\neq0}{\dfrac{x y}{x^{2}+y^{2}}}=\lim\limits_{y\to0,x\neq0}{\dfrac{x y}{x^{2}+y^{2}}}=0\) ,也连续。

但对二元函数 \(f\) ,若沿 \(y=kx\) 直线逼近,则 \(\lim\limits_{y=x,x\to0}{\dfrac{x y}{x^{2}+y^{2}}}=\dfrac{k}{1+k^2}\neq0,k\neq0\) 。因此若 \((0,0)\in[a, b] \times[c, d]\)\(f\)\((0,0)\) 处不连续。因此两个命题之间不能推断。

  1. \(f\left(\cdot, y_{0}\right):[a, b] \rightarrow \mathbb{R}\) 连续,对 \((x_0,y_0)\in[a, b] \times[c, d]\)\(\forall \varepsilon>0\)\(\exists \delta>0\) 使 \(\forall |x-x_0|<\delta-\delta(\dfrac{\varepsilon}{2})\) ,都有 \(|f(x,y_0)-f(x_0,y_0)|<\dfrac{\varepsilon}{2}\) 。因此 \(\exists\delta>0\) 使 \(\forall ||(x,y)-(x_0,y_0)||\leq||(x,y)-(x_0,y_0)||_{\infty}<\delta\) ,都有 \(\left|f(x, y)-f(x_0, y_{0})\right|\leq\left|f(x, y)-f(x, y_{0})\right|+|f(x,y_0)-f(x_0,y_0)|<\varepsilon\)

\(f\)\(\left(x_{0}, y_{0}\right)\) 连续。

  1. \(\forall (x_0,y_0)\in[a, b] \times[c, d]\)\(\forall \varepsilon>0\) ,由 \(f(x,y)\)\(x\)\(y\) 的连续性可以知道, \[ \begin{gathered} \left|f\left(x, y_{0}\right)-f\left(x_{0}, y_{0}\right)\right|<\varepsilon / 2\text{ \quad for\quad}\left|x-x_{0}\right| \leq\delta\\ \left|f\left(x_{0}+\delta, y\right)-f\left(x_{0}+\delta, y_{0}\right)\right|<\varepsilon / 2\text{ \quad for\quad}\left|y-y_{0}\right| \leq \eta_{1}\\ \left|f\left(x_{0}-\delta, y\right)-f\left(x_{0}-\delta, y_{0}\right)\right|<\varepsilon / 2\text{ \quad for\quad}\left|y-y_{0}\right| \leq \eta_{2} \end{gathered} \] 其中 \(\delta,\eta_1,\eta_2>0\) 。若取 \(\eta=\min(\eta_1,\eta_2)\) ,则对 \(\forall(x,y)\in[x_0-\delta,x_0+\delta]\times[y_0-\eta,y_0+\eta]\) ,均有 \[ \begin{aligned} &\quad \, \, {\left[f\left(x_{0}-\delta, y\right)-f\left(x_{0}-\delta, y_{0}\right)\right]+\left[f\left(x_{0}-\delta, y_{0}\right)-f\left(x_{0}, y_{0}\right)\right] } \\ & \leq f(x, y)-f\left(x_{0}, y_{0}\right) \\ & \leq\left[f\left(x_{0}+\delta, y\right)-f\left(x_{0}+\delta, y_{0}\right)\right]+\left[f\left(x_{0}+\delta, y_{0}\right)-f\left(x_{0}, y_{0}\right)\right] \end{aligned} \] 也就是 \(-\varepsilon<f(x,y)-f(x_0,y_0)<\varepsilon\) 。因此 \(f\) 关于 \((x,y)\) 连续。

1.4.7

\(U \subseteq \mathbb{R}^{2}\) 是开集, \((a, b) \in U, f\)\(U \backslash\{(a, b)\}\) 上有定义, 且满足极限 \[ A=\lim _{(x, y) \rightarrow(a, b)} f(x, y), \quad B=\lim _{y \rightarrow b} \lim _{x \rightarrow a} f(x, y) \] 都存在。证明 \(A=B\) 。由此知道, 如果 \[ \lim _{y \rightarrow b} \lim _{x \rightarrow a} f(x, y) \neq \lim _{x \rightarrow a} \lim _{y \rightarrow b} f(x, y) \] 则极限 \(\lim\limits _{(x, y) \rightarrow(a, b)} f(x, y)\) 不存在。


证明:极限都存在,因此对 \(\forall (x,y)\in U \backslash\{(a, b)\},\forall \varepsilon>0,\exists \delta>0\) ,使对 \(\forall0<||(x,y)-(a,b)||<\delta,|f(x,y)-A|<\varepsilon\) ,以及 \(\exists \eta_2>0\) 使对 \(\forall\varepsilon>0,\exists \eta_1>0\) \(\forall 0<|x-a|<\eta_1,\forall0<|y-b|<\eta_2,|f(x,y)-B|<\varepsilon\)

那么对 \(\forall 0<||(x,y)-(a,b)||<\min(\delta,\eta_1,\eta_2)\) ,则同时有 \(|f(x,y)-A|<\varepsilon,|f(x,y)-B|<\varepsilon\) 。若 \(A\neq B\),只要 \(\varepsilon<\dfrac{|A-B|}{2}\) 就会矛盾。因此 \(A=B\)

1.4.8

设 (a) 极限 \(H(y)=\lim\limits _{x \rightarrow a} f(x, y)\) 对任意 \(y \neq b\) 存在;(b) 极限 \(G(x)=\lim\limits _{y \rightarrow b} f(x, y)\) 对任意 \(x\) 存在, 且存在 \(\delta_{0}>0\) 使得:对任意 \(\varepsilon>0\), 存在 \(\delta_{2}(\varepsilon)>0\) 使得对任意 \(0<|y-b|<\delta_{2}(\varepsilon)\) 以及任意 \(0<|x-a|<\delta_{0}\) 都有 \(|f(x, y)-G(x)|<\varepsilon\) ; 则极限 \(\lim\limits _{x \rightarrow a} \lim\limits _{y \rightarrow b} f(x, y), \lim\limits _{y \rightarrow b} \lim\limits _{x \rightarrow a} f(x, y)\)\(\lim\limits _{x \rightarrow a, y \rightarrow b} f(x, y)\) 都存在, 且 \[ \lim _{x \rightarrow a, y \rightarrow b} f(x, y)=\lim _{y \rightarrow b} \lim _{x \rightarrow a} f(x, y)=\lim _{x \rightarrow a} \lim _{y \rightarrow b} f(x, y) . \]


证明:(1) 证明三个极限存在。

因极限 \(H(y)=\lim\limits _{x \rightarrow a} f(x, y)\) 对任意 \(y \neq b\) 存在,因此对任意 \(y \neq b\) 和任意 \(\varepsilon>0\), 存在 \(\delta_{1}(\dfrac{\varepsilon}{4})>0\) 使得对任意 \(0<|x-a|<\delta_{1}(\dfrac{\varepsilon}{4})\) 都有 \(|f(x, y)-H(y)|<\dfrac{\varepsilon}{4}\) 。因此,对任意 \(0<|x_{1,2}-a|<\min(\delta_{0},\delta_1(\dfrac{\varepsilon}{4}))\) 以及任意 \(0<|y-b|<\delta_{2}(\dfrac{\varepsilon}{4})\) 均有 \(|f(x_1,y)-f(x_2,y)|<|f(x_1, y)-H(y)|+|f(x_2, y)-H(y)|<\dfrac{\varepsilon}{2}\)

由题意知,存在 \(\delta_{0}>0\) 使得:对任意 \(\varepsilon>0\), 存在 \(\delta_{2}(\varepsilon)>0\) 使得对任意 \(0<|y-b|<\delta_{2}(\dfrac{\varepsilon}{4})\) 以及任意 \(0<|x_{1,2}-a|<\delta_{0}\) ,有 \(|G(x_1)-G(x_2)|\leq|f(x_1, y)-G(x_1)|+|f(x_2, y)-G(x_2)|+|f(x_1,y)-f(x_2,y)|<\varepsilon\) ,由 Cauchy 收敛定则,\(G(x)\) 在此处一定收敛于一个值 \(A\) ,因此 \(\lim\limits _{x \rightarrow a} \lim\limits _{y \rightarrow b} f(x, y)=A\)

同理,对任意 \(\varepsilon>0\),任意 \(0<|x-a|<\min(\delta_{0},\delta_1(\dfrac{\varepsilon}{4}))\) 以及任意 \(0<|y-b|<\delta_{2}(\dfrac{\varepsilon}{4})\)\(|f(x,y_1)-f(x,y_2)|<|f(x, y_2)-G(x)|+|f(x, y_1)-G(x)|<\dfrac{\varepsilon}{2}\) ,有 \(|H(y_1)-H(y_2)|\leq|f(x,y_1)-H(y_1)|+|f(x,y_2)-H(y_2)|+|f(x,y_1)-f(x,y_2)|<\varepsilon\) ,由 Cauchy 收敛定则,\(H(y)\) 在此处一定收敛于一个值 \(B\) ,因此 \(\lim\limits _{y \rightarrow b} \lim\limits _{x \rightarrow a} f(x, y)=B\)

对任意 \(\varepsilon>0\),任意 \(0<|x-a|<\min(\delta_{0},\delta_1(\dfrac{\varepsilon}{4}))\) 以及任意 \(0<|y-b|<\delta_{2}(\dfrac{\varepsilon}{4})\) ,即 \(||(x,y)-(a.b)||<\min[\delta_{0},\delta_1(\dfrac{\varepsilon}{4}),\delta_{2}(\dfrac{\varepsilon}{4})]\) ,有\(|f(x_1,y_1)-f(x_2,y_2)|\leq|f(x_1,y_1)-f(x_1,y_2)|+|f(x_1,y_2)-f(x_2,y_2)|<\varepsilon\) ,由 Cauchy 收敛定则,\(f(x, y)\) 在此处一定收敛于一个值 \(C\) ,因此 \(\lim\limits _{x \rightarrow a, y \rightarrow b} f(x, y)=C\)

  1. 证明它们相等。

1.4.7中已有证明加上对称性即可证明三者相等。

1.4.9

\[ f(x, y)= \begin{cases}x \sin \frac{1}{y}, & y \neq 0 \\ 0, & y=0\end{cases} \] 讨论极限 \(\lim\limits _{(x, y) \rightarrow(0,0)} f(x, y), \quad \lim\limits _{y \rightarrow 0} \lim\limits _{x \rightarrow 0} f(x, y)\) 以及 \(\lim\limits _{x \rightarrow 0} \lim \limits_{y \rightarrow 0} f(x, y)\)


解:有 \[ \lim_{y\to0}f(x,y)\left\{\begin{array}{cc} &不存在&,x\neq0\\ &=0&,x=0 \end{array}\right. \quad ,\lim_{x\to0}f(x,y)=0 \] 因此 \(\lim\limits _{y \rightarrow 0} \lim\limits _{x \rightarrow 0} f(x, y)=0\)\(\lim\limits _{x \rightarrow 0} \lim \limits_{y \rightarrow 0} f(x, y)\) 不存在。

\(\forall \varepsilon>0\) ,只要 \(||(x,y)||<\delta(\varepsilon)=\varepsilon\) ,就有 \(|f(x,y)|\leq|x|\leq||(x,y)||<\varepsilon\) ,因此 \(\lim\limits _{(x, y) \rightarrow(0,0)} f(x, y)=0\)

1.5.1

判断以下极限的存在性;若存在,求出极限值。

解:(1) \[ \lim _{(x, y) \rightarrow(0,0)} \frac{\arcsin \left(x^{2}+y^{2}\right)}{x^{2}+y^{2}} \]\(r=x^2+y^2\) ,显然 \(\lim\limits_{(x,y)\to(0,0)}{x^2+y^2}=0\) 。又因当 \((x,y)\to(0,0)\)\(x^2+y^2\neq0\) ,且 \(\lim \limits_{r\to0} \dfrac{\arcsin \left(r\right)}{r}=1\) ,故 \(\lim\limits _{(x, y) \rightarrow(0,0)} \dfrac{\arcsin \left(x^{2}+y^{2}\right)}{x^{2}+y^{2}}=1\)

(2) \[ \lim _{(x, y) \rightarrow(0,0)}\left(x^{2}+y^{2}\right) \mathrm{e}^{x-y} \]\(\left(x^{2}+y^{2}\right) \mathrm{e}^{x-y}\)\((0,0)\) 处是连续的,\(\lim\limits _{(x, y) \rightarrow(0,0)}\left(x^{2}+y^{2}\right) \mathrm{e}^{x-y}=0\)

(3) \[ \lim _{(x, y) \rightarrow(0,0)} \frac{x+y}{|x|+|y|} \] 沿射线 \(y=x,x>0\) 逼近,\(\lim\limits_{x\to0^+,y=x}{\dfrac{x+y}{|x|+|y|}}=1\) 。沿射线 \(y=-x,x>0\) 逼近,\(\lim\limits_{x\to0^+,y=-x}{\dfrac{x+y}{|x|+|y|}}=0\neq1\) 。因此极限不存在。

(4) \[ \lim _{(x, y) \rightarrow \infty} \frac{x+y}{x^{2}+x y+y^{2}} \] \((x,y)\to+\infty\) 意味着 \(\forall N>0\) 都有 \(r:=x^2+y^2>N\) 。有 \(|x+y|\leq|x|+|y|\leq2\sqrt{r}\)\(|xy|\leq|x||y|=\dfrac{1}{2}\sqrt{(|x|+|y|)^2-r}\leq\dfrac{\sqrt{3}}{2}r\) ,故 \[ |\frac{x+y}{x^{2}+x y+y^{2}}|=\frac{|x+y|}{|x^{2}+x y+y^{2}|}\leq\frac{2\sqrt{r}}{|x^2+y^2|-|xy|}\leq(8+4\sqrt{3})\frac{1}{\sqrt{N}} \]\(N\to\infty\) ,即可得 \(\lim\limits _{(x, y) \rightarrow \infty} \dfrac{x+y}{x^{2}+x y+y^{2}}=0\)

(5) \[ \lim _{(x, y) \rightarrow \infty}\left(\frac{|x y|}{x^{2}+x y+y^{2}}\right)^{x^{2}} \]

沿曲线 \(y=x\mathrm{e}^{\frac{k}{x^2}},x\to0^+\) 逼近(其中 \(k>0\) ),有 \(\dfrac{x}{y}=\mathrm{e}^{-\frac{k}{x^2}}<1\) ,因此 \[ \lim\limits_{x\to0^+}\left(\frac{|x y|}{x^{2}+x y+y^{2}}\right)^{x^{2}}=\lim\limits_{x\to0^+}\left(\frac{\mathrm{e}^{-\frac{k}{x^2}}}{1+\mathrm{e}^{-\frac{k}{x^2}}+\mathrm{e}^{-\frac{2k}{x^2}}}\right)^{x^2}=\mathrm{e}^{-k} \]

\(k\) 取不同的值时函数趋近值不同,因此极限不存在。

1.5.5

\(\Omega \subseteq \mathbb{R}^{m}\) 是一个非空集合。 (a) 证明函数 \(f: \mathbb{R}^{m} \rightarrow \mathbb{R}, f(\mathbf{x})=\inf \limits_{\mathbf{y} \in \Omega}\|\mathbf{y}-\mathbf{x}\|\),是一个连续函数。 (b) 若 \(\Omega\) 是闭集,证明对任意 \(\mathbf{x}\),存在 \(\mathbf{y}^{*} \in \Omega\) 使得 \(f(\mathbf{x})=\left\|\mathbf{y}^{*}-\mathbf{x}\right\|\) ,并且 \(\mathbf{y}^{*} \in \partial \Omega\) (即 \(\mathbf{y}^{*}\) 的任意邻域中都有不属于 \(\Omega\) 的点)。 (c) 若 \(\Omega_{1}, \Omega_{2}\) 是闭集且其中至少一个有界,证明存在 \(\mathbf{x}_{k}^{*} \in \Omega_{k}\) 使得 \[ \left\|\mathbf{x}_{1}^{*}-\mathbf{x}_{2}^{*}\right\|=\inf _{\mathbf{x}_{1} \in \Omega_{1}, \mathbf{x}_{2} \in \Omega_{2}}\left\|\mathbf{x}_{1}-\mathbf{x}_{2}\right\| \] (d) 举例说明上述 “闭”、“有界”的条件是不能或缺的。

\(\inf\) 是下确界函数)


解:(1)取 \(\forall \mathbf{x},\mathbf{y}\in\mathbb{R}^m,\forall \mathbf{p}\in \Omega\) ,由范数的三角形不等式:\(||\mathbf{y}-\mathbf{x}||\geq||\mathbf{x}-\mathbf{p}||-||\mathbf{y}-\mathbf{p}||\) ,可知 \(f(\mathbf{x})=\inf \limits_{\mathbf{p} \in \Omega}\|\mathbf{p}-\mathbf{x}\|\leq||\mathbf{y}-\mathbf{x}||+||\mathbf{y}-\mathbf{p}||\) ,进而 \(||\mathbf{y}-\mathbf{p}||\geq f(\mathbf{x})-||\mathbf{y}-\mathbf{x}||\)\(\forall \mathbf p\in\Omega\) 成立,因此 \(f(\mathbf{x})-f(\mathbf{y})\leq||\mathbf{y}-\mathbf{x}||\) 。同理 \(f(\mathbf{y})-f(\mathbf{x})\leq||\mathbf{y}-\mathbf{x}||\)

\(\forall \varepsilon>0\)\(\forall\mathbf{x}_1,\mathbf{x}_2\in\mathbb{R}^m\) ,只要 \(||\mathbf{x}_1-\mathbf{x}_2||<\varepsilon\) ,则 \(|f(\mathbf{x}_1)-f(\mathbf{x}_2)|\leq||\mathbf{x}_1-\mathbf{x}_2||<\varepsilon\) ,因此 \(f(\mathbf{x})\) 连续。

(2)对 \(\forall r>f(\mathbf{x})\) ,显然有 \(\inf \limits_{\mathbf{y} \in \Omega}\|\mathbf{y}-\mathbf{x}\|=\inf \limits_{\mathbf{y} \in \Omega\cap B(\mathbf{x},r)}\|\mathbf{y}-\mathbf{x}\|\) ,其中 \(B(0,r)=\{\mathbf{z}\in\mathbb{R}^m|||\mathbf{z}-\mathbf{x}||\leq r\}\) ,显然也是闭集(这是第一次作业第一题),因此 \(\Omega\cap B(0,r)\) 是有界闭集。又因 \(g(\mathbf{y})=||\mathbf{y}-\mathbf{x}||\) 连续,则 \(g(\mathbf{y}),\mathbf{y}\in\Omega\cap B(0,r)\) 也是一个有界闭集,因此有最小值 \(||\mathbf{y}_0-\mathbf{x}||\) 。由下确界定义,此最小值就是 \(\inf \limits_{\mathbf{y} \in \Omega\cap B(\mathbf{x},r)}\|\mathbf{y}-\mathbf{x}\|=\inf \limits_{\mathbf{y} \in \Omega}\|\mathbf{y}-\mathbf{x}\|\) 。因此 \[ \inf \limits_{\mathbf{y} \in \Omega}\|\mathbf{y}-\mathbf{x}\|=||\mathbf{y}_0-\mathbf{x}||,\quad \mathbf{y}_0\in\Omega\cap B(0,r) \]\(\forall\varepsilon>0\) ,有 \(\mathbf{z}=\mathbf{y}_0-\dfrac{\varepsilon}{2}\dfrac{\mathbf{y}_0-\mathbf{x}}{||\mathbf{y}_0-\mathbf{x}||}\)\(||\mathbf{y}_0-\mathbf{z}||=\dfrac{\varepsilon}{2}<\varepsilon\) ,但 \(||\mathbf{z}-\mathbf{x}||=f(\mathbf{x})(1-\dfrac{\varepsilon}{2})<f(\mathbf{x})\) ,因此 \(\mathbf{z}\notin\Omega\) 。所以 \(\mathbf{y}_0\in\partial\Omega\)

(3)不妨设 \(\Omega_1\) 是有界闭集。对 \(\forall\mathbf{x}\in\Omega_2 \ ,\ \exists\mathbf{y}_0\in\Omega_1\ ,\ f(\mathbf{x}):=\inf\limits_{\mathbf{y}\in\Omega_1}{||\mathbf{y}-\mathbf{x}||}=||\mathbf{y}_0-\mathbf{x}||\) 。对 \(\forall n\in\Z^+\) 存在 \(\mathbf{x}_n\in\Omega_2\) ,使其中存在收敛子列 \(\{\mathbf{x}_{n_k}\}\) 使 \(\lim\limits_{k\to+\infty}{||f(\mathbf{x}_{n_k})||}=\inf\limits_{\mathbf{x}\in\Omega_2}{f(\mathbf{x})}\) ,因 \(f\) 为连续映射且 \(\Omega_2\) 是闭集,存在 \(\mathbf{x}_0\in\Omega_2\) 使 \(f(\mathbf{x}_0)=\inf\limits_{\mathbf{x}\in\Omega_2}{f(\mathbf{x})}\)​ 。因此存在 \(\mathbf{x}_{k}^{*} \in \Omega_{k}\) 使得 \[ \left\|\mathbf{x}_{1}^{*}-\mathbf{x}_{2}^{*}\right\|=\inf _{\mathbf{x}_{1} \in \Omega_{1}, \mathbf{x}_{2} \in \Omega_{2}}\left\|\mathbf{x}_{1}-\mathbf{x}_{2}\right\| \] (4)没有“闭”限制:

\(\Omega_1=(-1,0),\Omega_2=(0,1)\) ,显然不存在 \(x_k\in\Omega_k,|x_1-x_2|=\inf\limits_{x_1\in\Omega_1,x_2\in\Omega_2}{|x_1-x_2|}=0\)

没有“有界”限制:

\(\Omega_1=\{(x,y)\in\R^2|y\geq\dfrac{1}{x},x>0\},\Omega_2=\{(x,y)\in\R^2|y\leq-\dfrac{1}{x},x>0\}\) 。显然不存在 \((x_k,y_k)\in\Omega_k,||(x_1,y_1)-(x_2,y_2)||=\inf\limits_{x_1\in\Omega_1,x_2\in\Omega_2}{||(x_1,y_1)-(x_2,y_2)||}=0\)